Transcript for:
Anatomy, Physiology and Pharmacology Overview

This is made by my friend and me. If you have any questions or comments, please feel free to contact me at [email protected]. Thank you. The file will be updated hopefully. ________________ Table of contents Anatomy & Histology 4 HNS02 Meninges and blood supply 5 HNS03 Brainstem 7 HNS04 Cranial nerves 9 HNS07 Thalamus and cerebral cortex 11 HNS19 Structure and function of head and neck 12 HNS20 Vasculature and lymphatic of head and neck 15 HNS21 Nerves of head and neck 17 HNS22 Face and scalp 18 HNS23 Nasal cavity 22 HNS24 Oral cavity, submandibular and sublingual glands 25 HNS27 Pharynx 27 HNS28 Parotid gland and infratemporal fossa 28 HNS29 Larynx 30 HNS37 Orbit 32 HNS40 Ear 34 Physiology 39 HNS01 Overview of the central nervous system 40 HNS12 Somaesthetic pathway and somatic sensation 41 HNS13 Pain pathway and mechanisms of pain 43 HNS15 Motor system 44 HNS16 Cerebellum 46 HNS17 Control of movement 48 HNS26 Salivary and nasal secretion and their effects on taste and smell 49 HNS30 Swallowing and speech 51 HNS38 Vision 52 HNS39 Eye movements 53 HNS41 Audition 55 HNS42 Vestibular system 56 HNS43 Higher cortical function 57 HNS46 Limbic system, learning and memory 62 HNS47 Behavioral neuroscience 63 Pathology 66 HNS11 Pathology of CNS infection 67 HNS18 Pathology of raised intracranial pressure and cerebrovascular disease 70 HNS32 Pathology of intracranial tumours 73 HNS33 Common ENT cancers: Anatomic and physiologic appraisal 74 HNS34 Common ENT inflammatory diseases: Anatomic and physiologic appraisal 75 HNS35 NPC and other head and neck tumours 76 Microbiology 77 HNS05 Infection of the CNS I: Bacterial and fungal infections 78 HNS06 Infections of the CNS II: Viral and prion infections of the CNS 82 HNS31 Infectious causes of upper airway obstruction 83 Pharmacology 84 HNS10 Infections of the CNS III: Principles of antiviral therapy 85 HNS14 Narcotic analgesics 86 HNS25 Sedatives and hypnotics 87 HNS36 Autonomic nervous system 88 HNS44 Antidepressants 89 HNS45 Antipsychotics 91 HNS50 Drugs used for neurodegenerative diseases 92 Biochemistry 93 HNS08 Chemical neurotransmission I 94 HNS09 Chemical neurotransmission II 96 Molecular medicine 97 HNS48 Neurodegenerative diseases 98 HNS49 Molecular basis of neurological diseases 99 Public health 100 HNS51 When we all become Methuselah 101 HNS52 Impacts of evidence-based versus defensive medical practices: To CT or not to CT in traumatic head injury 102 Others 103 ________________ Anatomy & Histology ________________ HNS02 Meninges and blood supply [2020 M24 2nd Summative Q21] A junior doctor was asked to collect a sample of cerebrospinal fluid from a 30-year-old patient for analysis. Where should he insert the spinal needle? A. Epidural space at L3/L4 interspinous level B. Intervertebral foramen between L3 and L4 C. Subarachnoid space at L3/L4 interspinous level D. Subdural space at L3/L4 interspinous level Ans: C [2021 M25 2nd Summative Q58] The middle cerebral artery (MCA) is a major source of blood supply to the brain. Which of the following would MOST LIKELY occur following an acute MCA occlusion? A. Cortical blindness B. Double vision C. Speech disorder D. Unsteady gait Ans: C [2022 M26 2nd Summative Q42] A 45-year-old male came to the clinic due to oculomotor nerve palsy. After examination, the clinician concluded that his symptoms were caused by compression. The aneurysm of which artery MOST LIKELY compressed the root of the oculomotor nerve? A. Anterior inferior cerebellar artery B. Basilar artery C. Posterior inferior cerebellar artery D. Superior cerebellar artery Ans: D Explanation / reference to students: Superior cerebellar artery arises near the end of the basilar artery, passing laterally around the brainstem. Its course is immediately inferior to the oculomotor nerve. Therefore, an aneurysm of superior cerebellar artery causes compression on oculomotor nerve leading to the nerve palsy. ________________ [2023 M27 2nd Summative Q19] A junior doctor was asked to collect a sample of cerebrospinal fluid from a 30-year-old patient for analysis. Where should he insert the spinal needle? A. Epidural space at L3/L4 interspinous level B. Intervertebral foramen between L3 and L4 C. Subarachnoid space at L3/L4 interspinous level D. Subdural space at L3/L4 interspinous level Ans: C [2023 M27 2nd Summative Q58] Occlusion of the internal carotid artery will diminish blood supply to the brain regions supplied by it. Which of the following structures is MOST LIKELY to be affected? A. Calcarine gyrus B. Cerebellar hemisphere C. Inferior frontal gyrus D. Pons Ans: C [2024 M28 Sample Questions] A 45-year-old male came to the clinic due to oculomotor nerve palsy. After examination, the clinician concluded that his symptoms were caused by compression. The aneurysm of which artery MOST LIKELY compressed the root of the oculomotor nerve? A. Anterior inferior cerebellar artery B. Basilar artery C. Posterior communicating artery D. Superior cerebellar artery Ans: D Explanation / reference to students: Superior cerebellar artery arises near the end of the basilar artery, passing laterally around the brainstem. Its course is immediately inferior to the oculomotor nerve. Therefore, an aneurysm of superior cerebellar artery causes compression on oculomotor nerve leading to the nerve palsy. ________________ HNS03 Brainstem [2019 M23 2nd Summative Q43] Which brain area is regulated by the neurotransmitter serotonin? A. Hypoglossal nucleus B. Locus coeruleus C. Raphe nucleus D. Substantia nigra E. Vestibular nucleus Ans: C [2023 M27 2nd Summative Q93] Which nucleus of the brain is mediated by neurotransmitter serotonin? A. Basal nucleus of Meynert B. Locus coeruleus C. Raphe nucleus D. Vestibular nucleus Ans: C Option A: Acetylcholine Option B: Norepinephrine [2021 M25 2nd Summative Q41] A 50-year-old man is diagnosed with Parkinson disease. The patient has undergone positron emission tomography scan examination for dopaminergic neurons. Which brain region shows loss of dopaminergic neurons? A. Medial longitudinal fasciculus B. Periaqueductal gray C. Substantia nigra D. Superior colliculus Ans: C Explanation / reference to students: This is a question for the neuroanatomy of Parkinson disease. Students should know the most populated regions with dopaminergic neurons. Option C is the answer. ________________ [2024 M28 Sample Questions] A 50-year-old man is diagnosed with Parkinson disease. The patient has undergone positron emission tomography scan examination for dopaminergic neurons. Which brain region is MOST LIKELY to show loss of dopaminergic neurons? A. Medial longitudinal fasciculus B. Periaqueductal gray C. Substantia nigra D. Superior colliculus Ans: C Explanation / reference to students: This is a question for the neuroanatomy of Parkinson disease. Students should know the most populated regions with dopaminergic neurons. Option C is the answer. [2024 M28 Sample Questions] The cerebral peduncle is a region of the midbrain. Which of the following structure passes through the cerebral peduncle? A. Corticospinal tract B. Medial lemniscus C. Medial longitudinal fasciculus D. Spinothalamic tract Ans: A Explanation / reference to students: This is a question for general neuroanatomy. This is a major concept of how motoneurons are connected from the cerebral cortex to the spinal cord. Motoneurons in descending pathway go through cerebral peduncle. Only option A is the correct answer. The other three options are part of the ascending sensory neurons. ________________ HNS04 Cranial nerves [2019 M23 2nd Summative Q40] After tonsillectomy, a boy experienced absence of tactile and taste sensation in the posterior 1/3 of the tongue. Which nerve was inadvertently severed during tonsillectomy? A. Chorda tympani B. Glossopharyngeal nerve C. Lingual nerve D. Spinal accessory nerve E. Vagus nerve Ans: B CN IX: Inferior (petrosal) ganglion (special sensory) → from solitary nucleus to taste buds in posterior ⅓rd of tongue → taste [2020 M24 2nd Summative Q35] The pterygopalatine ganglion plays a major function in the nasal cavity and surrounding regions. What is the MOST COMMON sign manifested if maxillary tumours spread to the medial wall and invade this ganglion? A. Double vision B. Dryness of the nasal walls C. Excessive secretion of tears D. Hyposecretion of tears Ans: C Comment from Question Writer * This question was designed for critical thinking and application * The pterygopalatine ganglion is one of the four parasympathetic ganglia in the head and neck. Although it contains sympathetic, sensory and special visceral sensory (taste) fibres, the parasympathetic function still predominates. Affectation during disease process would likely produce a parasympathetic response rather than sympathetic. This would lead to vasodilation of blood vessels and increased secretions of glands. The increased tear secretion is a function of the parasympathetic fibre through the greater petrosal nerve (CN VII) of this ganglion (refers to answer C). On this basis, the answers B and D are not the best answers. * A answer has nothing to do with the function of the ganglion since extraocular muscles are supplied by CNIII, CN IV and VI. ________________ [2022 M26 2nd Summative Q43] A 30-year-old female was diagnosed with skull base meningioma compressing at structure A as shown in the diagram below. (Source: Anatomy - A Photographic Atlas: Arteries and the Arterial Circle of Willis) Which muscle movement does the patient have difficulties with on the right side? A. Directing the pupil laterally B. Elevating the upper eyelid C. Forcibly closing the eyelids D. Pupillary constriction Ans: A ________________ HNS07 Thalamus and cerebral cortex [2020 M24 2nd Summative Q58] Homeostasis is the maintenance of a constant internal environment. Which brain region is primarily responsible for homeostasis? A. Amygdala B. Hippocampus C. Hypothalamus D. Thalamus Ans: C [2022 M26 2nd Summative Q44] There are many connections of different brain regions in the Papez circuit. Which nucleus of the thalamus connects to the mammillary bodies via the mammillothalamic tracts? A. Anterior nucleus B. Ventral anterior nucleus C. Ventral lateral nucleus D. Ventral posterior nucleus Ans: A [2024 M28 Sample Questions] Mr. Chan is a 70-year-old gentleman who has recently suffered from cerebrovascular accident and has been comatose for 4 days. After he has woken up from the coma, his relatives discover that he has difficulty in producing intelligible speech. His ability to understand speech and his voice are all normal. Which of the following structures is LIKELY to be damaged by the cerebrovascular accident? A. Left inferior frontal gyrus B. Left internal capsule C. Left vagal nucleus D. Right cerebellopontine angle Ans: A Explanation / reference to students: Option A: Left inferior frontal gyrus is for language production. Option B: Damaged left internal capsule leads to contralateral hemiparesis. Option C: Damaged vagus nucleus leads to gastroparesis, swallowing and phonation problems. Option D: Cerebellopontine angle syndrome includes unilateral hearing loss, vertigo, facial nerve palsy and numbness. ________________ HNS19 Structure and function of head and neck [2022 M26 2nd Summative Q37] A boy started to visibly contract muscles in the neck during an asthma exacerbation. Which of the following muscles is contracted during breathing in this boy? (Source : McMinn's and A Brahams' Atlas of Clinical Anatomy, 7th edition) A. A B. B C. C D. D Ans: D Explanation / reference to students: 1. Notice that the sternocleidomastoid muscle has been removed in the specimen. Therefore, the contents of the carotid sheath are exposed, such as the carotid arteries and internal jugular vein. We can use these structures to estimate the outline of the anterior and posterior triangles of the neck. 2. Notice that, in the anterior triangle of the neck, we can see supra- and infrahyoid muscles, including: Muscle A – Sternohyoid; Muscle B – Omohyoid (with two bellies); Muscle C – Sternothyroid (superior attachment is hidden deep to the superior belly of omohyoid). 3. Notice that, in the posterior triangle of the neck, we can observe deep muscles of the neck, including the anterior scalene muscle (D), descending to the 1st rib, in front of the brachial plexus. 4. Remember that the scalene muscles (not the infrahyoid muscles) are considered accessory muscles of inspiration. ________________ [2023 M27 2nd Summative Q41] Lateral view of head and neck. Wbrahans Pal ; Spratt LD. , Loukas M.; Van Schoor A.N. (2019). Abrahams' and Mcminn's clinical atlas of human anatomy (8" ed.). Elsevier. Which nerve(s) crosses the same skull base opening as the arrowed vessel? A. Accessory nerve B. Hypoglossal nerve C. Phrenic nerve D. Sympathetic nerves Ans: A [2024 M28 Sample Questions] A boy started to visibly contract muscles in the neck during an asthma exacerbation. Which of the following muscles is contracted during breathing in this boy? (Source: McMinn's and Abrahams' Atlas of Clinical Anatomy, 7th edition) A. A B. B C. C D. D Ans: D Explanation / reference to students: 1. Notice that the sternocleidomastoid muscle has been removed in the specimen. Therefore, the contents of the carotid sheath are exposed, such as the carotid arteries and internal jugular vein. We can use these structures to estimate the outline of the anterior and posterior triangles of the neck. 2. Notice that, in the anterior triangle of the neck, we can see supra- and infrahyoid muscles, including: Muscle A – Sternohyoid; Muscle B – Omohyoid (with two bellies); Muscle C – Sternothyroid (superior attachment is hidden deep to the superior belly of omohyoid). 3. Notice that, in the posterior triangle of the neck, we can observe deep muscles of the neck, including the anterior scalene muscle (D), descending to the 1st rib, in front of the brachial plexus. 4. Remember that the scalene muscles (not the infrahyoid muscles) are considered accessory muscles of inspiration. ________________ HNS20 Vasculature and lymphatic of head and neck [2021 M25 2nd Summative Q42] The head and neck regions receive the majority of their blood supply through the carotid and vertebral arteries. What is the arrowed artery? A. Common carotid B. External carotid C. Internal carotid D. Vertebral Ans: B [2023 M27 2nd Summative Q46] Venous drainage of the infratem poral fossa. Picture from Anatomy & Physiology Revealed, McGraw Hill What is the arrowed vein? A. Deep facial vein B. Internal jugular vein C. Retromandibular vein D. Superficial temporal vein Ans: C ________________ HNS21 Nerves of head and neck [2023 M27 2nd Summative Q90] A patient has Horner syndrome, with ptosis and miosis. The presynaptic sympathetic fibres that are affected MOST LIKELY travel through the white ramus of which spinal nerve? A. C2 B. Ll C.L5 D. T1 Ans: D ________________ HNS22 Face and scalp [2019 M23 2nd Summative Q49] A 30-year-old male sustained a left eye trauma during a football game. He was found to have orbital left floor fracture on imaging. Clinically, he had left hypoesthesia around his left upper cheek. Which of the following nerves is MOST LIKELY involved as a result of his injury? A. Facial nerve B. Infraorbital nerve C. Lacrimal nerve D. Trochlear nerve E. Zygomatic nerve Ans: B [2020 M24 2nd Summative Q34] Which muscle primarily contracts during smiling? A. Levator anguli oris B. Orbicularis oris C. Zygomaticus major D. Zygomaticus minor Ans: C Comment from Question Writer * Two main muscles are involved in an open, bright smile: zygomaticus major and risorius. * As the risorius was not mentioned, the only correct answer was C. * The levator anguli oris, is not as powerful as two other mentioned muscles and activation of this muscle will give an impression of a ‘fake smile’. [2020 M24 2nd Summative Q45] The infraorbital nerve can be damaged during an orbital blow-out fracture. Which of the following nerves gives rise to the infraorbital nerve? A. Facial nerve B. Maxillary nerve C. Nasociliary nerve D. Ophthalmic nerve Ans: B [2021 M25 2nd Summative Q34] A 24-year-old woman is brought to the Accident and Emergency Department after a motor vehicle collision. Radiological studies show a frontal bone fracture. Physical examination reveals a lack of sensation from the lateral aspect of the forehead. Which nerve is MOST LIKELY injured in this patient? A. Anterior ethmoidal B. Oculomotor C. Supraorbital D. Zygomatic Ans: D [2022 M26 2nd Summative Q39] A patient was diagnosed with a left-sided facial paresis and was asked to perform a teeth showing smile. The patient could not move the left lower lip while the movements of the left upper lip and the other facial muscles were intact. Which branch of the left facial nerve is injured? A. Buccal branch B. Marginal mandibular branch C. Temporal branch D. Zygomatic branch Ans: B [2022 M26 2nd Summative Q49] The patient has not undergone any dental work or has a history of dental caries but describes pain across left upper jaw as sharp, lancinating ("shooting"), and that the pain is aggravated by chewing, brushing teeth, and talking. Which nerve carries nociceptive impulses? A. Buccal branch of facial nerve B. Great auricular C. Maxillary D. Supratrochlear Ans: C Explanation / reference to students: The pain is likely to be neuropathic pain. The maxillary nerve is responsible for carrying sensory impulses from the hard palate. ________________ [2022 M26 2nd Summative Q50] Which pair of muscles are primarily involved in whistling? A. Depressor anguli oris and zygomaticus major B. Levator labii superioris and depressor labii inferioris C. Mentalis and platysma D. Orbicularis oris and buccinator Ans: D [2023 M27 2nd Summative Q36] A 24-year-old woman is brought to the Accident and Emergency Department after a motor vehicle collision. Radiological studies show a frontal bone fracture. Physical examination reveals a lack of sensation over the forehead. Which nerve is MOST LIKELY injured in this patient? A. Anterior ethmoidal B. Oculomotor C. Supraorbital D. Zygomatic Ans: C [2023 M27 2nd Summative EMQ Q7] VII. CUTANEOUS NERVES OF THE FACE AND SCALP A. Auriculotemporal nerve B. External nasal nerve C. Infraorbital nerve D. Greater auricular nerve E. Greater occipital nerve F. Lacrimal nerve G. Lesser occipital nerve H. Mental nerve I. Supraorbital nerve J. Supratrochlear nerve For each of the following regions, match the corresponding nerve in the list of options above that innervates the region. Each option may be used once, more than once, or not at all. Question 17 Vertex of the cranium Ans: I Supraorbital nerve Comment from Question Writer * It refers to sensory supply of the three branches of the trigeminal nerve to the face and anterior half of the scalp. The posterior half of the scalp and neck are supplied by the posterior and anterior rami of the spinal nerves (C2-C3). Question 18 Tragus of the ear Ans: A Auriculotemporal nerve [2024 M28 Sample Questions] The patient has not undergone any dental work or has a history of dental caries but describes pain across left upper jaw as sharp, lancinating ("shooting"), and that the pain is aggravated by chewing, brushing teeth, and talking. Which nerve carries nociceptive impulses? A. Buccal branch of facial nerve B. Great auricular C. Maxillary D. Supratrochlear Ans: C Explanation / reference to students: The pain is likely to be neuropathic pain. The maxillary nerve is responsible for carrying sensory impulses from the hard palate. ________________ HNS23 Nasal cavity [2020 M24 2nd Summative Q37] A patient manifested nasal free flow of clear fluid after sustaining a severe head trauma. Which bone is fractured? A. Ethmoid B. Frontal C. Maxillary D. Sphenoid Ans: A [2021 M25 2nd Summative Q37] The image below shows the lateral wall of the nasal cavity. Secretion from which cavity drains into A? A. Anterior ethmoid sinus B. Maxillary sinus C. Middle ethmoid sinus D. Sphenoid sinus Ans: C [2022 M26 2nd Summative Q46] The pterygopalatine ganglion has several branches which carry autonomic and sensory fibres to the nasal cavity. Which branch carries special visceral sensory fibres for taste to the roof of the oral cavity? A. Lesser palatine nerve B. Nasal branch C. Nasopalatine nerve D. Pharyngeal branch Ans: C ________________ [2021 M25 2nd Summative Q43] The image below shows the lateral wall of the nasal cavity. The opening of which structure is found in this slit-like structure? A. Anterior ethmoidal air cells B. Nasolacrimal duct C. Posterior ethmoidal air cells D. Sphenoid sinus Ans: A [2022 M26 2nd Summative Q38] Vidian neurectomy was used to treat a patient with chronic rhinitis and rhinorrhoea. With reference to the picture below, which opening transmits the nerve of the pterygoid canal? (Source: McMim's and Abrahams' Atlas of Clinical Anatomy, 7th edition) A. A B. B C. C D. D Ans: B Comment from Question Writer * A: Pharyngeal canal, between nasopharynx and pterygopalatine fossa. Transmits the pharyngeal branch of V2. * B: Pterygoid canal, between foramen lacerum and pterygopalatine fossa. Transmits the nerve of the pterygoid canal (Vidian). * C: Foramen ovale, between middle cranial fossa and infratemporal fossa. Transmits V3. * D: Foramen lacerum, between base of skull and middle cranial fossa. Normally, plugged with a cartilage which transmits the greater petrosal nerve, for example. Above it, is the superior part of the carotid canal. ________________ HNS24 Oral cavity, submandibular and sublingual glands [2019 M23 2nd Summative Q59] Flavour arises from a mixture of sensory inputs composed of smell, taste, and the tactile sensation of food upon chewing. What is the structure involved in the detection of tactile sensation of food upon chewing? A. Circumvallate papillae B. Dermal papillae C. Filiform papillae D. Foliate papillae E. Fungiform papillae Ans: C Out of syllabus [2021 M25 2nd Summative Q39] Extrinsic tongue muscles originate outside the tongue. Which extrinsic tongue muscle receives motor innervation different from all other extrinsic tongue muscles? A. Genioglossus B. Hyoglossus C. Palatoglossus D. Styloglossus Ans: C [2021 M25 2nd Summative Q35] A 54-year-old woman complained of sharp, lancinating ("shooting") pain across her left lower jaw, and that the pain was aggravated by chewing, brushing her teeth, and talking. Which nerve is MOST LIKELY involved? A. Facial B. Lingual C. Mandibular D. Maxillary Ans: C [2022 M26 2nd Summative Q41] Most muscles in the tongue are supplied by CN XII. Which extrinsic muscle of the tongue receives a different nerve supply? A. Genioglossus B. Hyoglossus C. Palatoglossus D. Styloglossus Ans: C ________________ [2024 M28 Sample Questions] Extrinsic tongue muscles originate outside the tongue. Which extrinsic tongue muscle receives motor innervation that is different from that of all other extrinsic tongue muscles? A. Genioglossus B. Hyoglossus C. Palatoglossus D. Styloglossus Ans: C Explanation / reference to students: The topic is: Oral cavity, submandibular and sublingual glands. This question is assessing the anatomy knowledge of the innervation of the extrinsic tongue muscle. Most of the muscles of the tongue are innervated by CNXII, with one exception. Palatoglossus is innervated by CNX. The correct answer is Option C. Options A, B and D are all innervated by CNXII. ________________ HNS27 Pharynx [2020 M24 2nd Summative Q36] A foreign body lodging at the piriform fossa causes irritation and coughing. What is the innervation at this site? A. External laryngeal nerve B. Glossopharyngeal nerve C. Internal laryngeal nerve D. Lingual nerve Ans: C [2023 M27 2nd Summative Q38] Stylopharyngeus muscle is one of the muscles of the pharynx. What is the function of this muscle? A. Acts as the sphincter of lower end of the pharynx B. Assists in closing the nasopharynx C. Depresses the pharynx D. Elevates the larynx during swallowing Ans: D ________________ HNS28 Parotid gland and infratemporal fossa [2021 M25 2nd Summative Q38] The mandibular nerve carries both motor and sensory fibres and gives off all the branches in the infratemporal fossa. The branch from which nerve provides motor innervation to the mylohyoid muscle? A. Auriculotemporal nerve B. Buccal nerve C. Inferior alveolar nerve D. Lingual nerve Ans: C [2022 M26 2nd Summative Q40] A patient was traumatically injured at the site of the pterion resulting in epidural haematoma. Which artery is MOST LIKELY affected in this region? A. Deep temporal artery B. Middle meningeal artery C. Superficial temporal artery D. Supraorbital artery Ans: B [2023 M27 2nd Summative Q37] A patient was traumatically injured at the site of the pterion resulting in an epidural haematoma. Which artery is MOST LIKELY affected in this region? A. Deep temporal artery B. Middle meningeal artery C. Superficial temporal artery D. Supraorbital artery Ans: B [2023 M27 2nd Summative Q44] Salivary stone can cause blockage of the salivary duct. Which muscle prevents the excretion of the stone from the parotid duct? A. Buccinator B. Masseter C. Orbicularis oris D. Zygomaticus major Ans: A [2023 M27 2nd Summative Q94] Lateral view of the left infratemporal fossa, with partial removal of the mandible. (Source: Logan's illustrated human anatomy: A pictorial introduction to basic form and structure. 1st Edition) What is the action of muscle X on the mandible? A. Depression and protrusion B. Depression and retrusion C. Elevation and protrusion D. Elevation and retrusion Ans: C ________________ HNS29 Larynx [2022 M26 2nd Summative Q47] The vestibule of the larynx is located above the true vocal cords. What is the sensory innervation of this region? A. External laryngeal nerve B. Inferior laryngeal nerve C. Internal laryngeal nerve D. Recurrent laryngeal nerve Ans: C Explanation / reference to students: Internal laryngeal nerve is a branch of superior laryngeal nerve which is sensory and mainly supplying structures above the vocal cords. [2022 M26 2nd Summative Q62] Miss Wong underwent left hemithyroidectomy 2 weeks ago and has recovered well. However, when she returned to work as a singing teacher, she found that she had difficulty in singing the soprano range. Other than that, she is able to speak normally and eat normally. What structure is MOST LIKELY to be damaged during the operation leading to the present symptom? A. External branch of left superior laryngeal nerve B. Left recurrent laryngeal nerve C. Left true vocal cord D. Left vagus nerve Ans: A Explanation / reference to students: Option A: External branch of superior laryngeal nerve innervates the cricothyroid muscle to length the vocal cord, to produce high pitched sound. Option B: Unilateral recurrent laryngeal nerve injury leads to unilateral vocal cord palsy, and patient will complain of hoarseness of voice (breathy voice) and choking. Option C: Injured vocal cord might cause hoarseness of voice, depends of the degree and mechanism of the injury. Option D: Injury of vagus nerve leads to hoarseness of voice and choking. [2023 M27 2nd Summative Q39] The vestibule of the larynx is located above the true vocal cords. What is the sensory innervation of this region? A. External laryngeal nerve B. Inferior laryngeal nerve C. Internal laryngeal nerve D. Recurrent laryngeal nerve Ans: C Explanation / reference to students: Internal laryngeal nerve is a branch of superior laryngeal nerve which is sensory and mainly supplying structures above the vocal cords. [2024 M28 Sample Questions] The vestibule of the larynx is located above the true vocal cords. What is the sensory innervation of this region? A. External laryngeal nerve B. Inferior laryngeal nerve C. Internal laryngeal nerve D. Recurrent laryngeal nerve Ans: C Explanation / reference to students: Internal laryngeal nerve is a branch of superior laryngeal nerve which is sensory and mainly supplying structures above the vocal cords. [2024 M28 Sample Questions] Miss Wong underwent left hemithyroidectomy 2 weeks ago and has recovered well. However, when she returned to work as a singing teacher, she found that she had difficulty in singing the soprano range. Other than that, she is able to speak normally and eat normally. What structure is MOST LIKELY to be damaged during the operation leading to the present symptom? A. External branch of left superior laryngeal nerve B. Left recurrent laryngeal nerve C. Left true vocal cord D. Left vagus nerve Ans: A Explanation / reference to students: Option A: External branch of superior laryngeal nerve innervates the cricothyroid muscle to length the vocal cord, to produce high pitched sound. Option B: Unilateral recurrent laryngeal nerve injury leads to unilateral vocal cord palsy, and patient will complain of hoarseness of voice (breathy voice) and choking. Option C: Injured vocal cord might cause hoarseness of voice, depends of the degree and mechanism of the injury. Option D: Injury of vagus nerve leads to hoarseness of voice and choking. ________________ HNS37 Orbit [2022 M26 2nd Summative Q48] Intorsion is an internal rotation of the eyeball. What are the muscles involved in this action? A. Inferior rectus muscle and inferior oblique B. Medial rectus muscle and inferior rectus muscle C. Medial rectus muscle and superior rectus muscle D. Superior rectus muscle and superior oblique muscle Ans: D Explanation / reference to students: Internal rotation of the eyeball is carried out by superior rectus and superior oblique muscles. [2023 M27 2nd Summative Q40] Intorsion is an internal rotation of the eyeball. What are the muscles involved in this action? A. Inferior rectus muscle and inferior oblique B. Medial rectus muscle and inferior rectus muscle C. Medial rectus muscle and superior rectus muscle D. Superior rectus muscle and superior oblique muscle Ans: D Explanation / reference to students: Internal rotation of the eyeball is carried out by superior rectus and superior oblique muscles. [2023 M27 2nd Summative EMQ Q2] II. MUSCLES OF THE EYE A. Ciliary B. Dilatator pupillae C. Inferior oblique D. Inferior rectus E. Lateral rectus F. Levator palpebrae superioris G. Medial rectus H. Sphincter pupillae I. Superior oblique J. Superior rectus For each of the following descriptions, match the corresponding muscle from the list of options above. Each option may be used once, more than once, or not at all. Q3 Depresses the eye from a starting adducted position Ans: I Q4 Adjacent to the frontal nerve (branch of CN V1) Ans: F Q5 Adjacent to the ciliary ganglion Ans: E Lateral rectus Comment from Question Writer * The ciliary ganglion is one of the four parasympathetic ganglia of the head. It lies in the orbit, sandwiched between the optic nerve and the lateral rectus muscle (option E). This relationship was discussed and, more importantly, visualized during the dissection of the orbit, as students dissected the orbit from a superior approach. * [2024 M28 Sample Questions] Intorsion is an internal rotation of the eyeball. What are the muscles involved in this action? A. Inferior rectus muscle and inferior oblique B. Medial rectus muscle and inferior rectus muscle C. Medial rectus muscle and superior rectus muscle D. Superior rectus muscle and superior oblique muscle Ans: D Explanation / reference to students: Internal rotation of the eyeball is carried out by superior rectus and superior oblique muscles. ________________ HNS40 Ear [2019 M23 2nd Summative Q41] The arcuate eminence is part of the petrous bone and located at the roof of the middle ear. What structure is found directly beneath the arcuate eminence? A. Anterior semicircular canal B. Cochlea and utricle C. Lateral semicircular canal D. Posterior semicircular canal E. Promontory Ans: A Comment from Question Writer * This was taught in Practical 4 of the Head and Neck Practical, Station No. 5. * In fact, I asked the same question in the student’s task of this station. The value of this question to test the anatomical relationship of the middle ear and inner ear. The arcuate eminence is a small upwards projection of the petrous temporal bone into the cranial cavity above. It marks the position of the anterior semicircular canal. [2020 M24 2nd Summative Q38] Equalisation of pressure in the middle ear depends on the function of two essential muscles in the pharynx. What are these muscles? A. Palatoglossus and salpingopharyngeus B. Palatopharyngeus and salpingopharyngeus C. Tensor veli palatini and levator veli palatini D. Tensor veli palatini and palatoglossus Ans: C ________________ [2021 M25 2nd Summative Q36] The image below shows the lateral view of the middle ear. What is the function of A? A. Nerve supply to the tensor veli palatini muscle B. Parasympathetic nerve supply to the parotid gland C. Sensory supply to the anterior 2/3 of the tongue D. Special visceral sensory supply to the anterior 2/3 of the tongue Ans: D ________________ [2021 M25 2nd Summative EMQ Q4] IV.STRUCTURES RELATED TO THE MIDDLE EAR A. Chorda tympani B. Fenestra cochlea C. Greater petrosal nerve D. Pharyngotympanic tube E. Processus cochleariformis F. Promontory G. Pyramid H. Tensor tympani I. Tympanic canaliculus J. Stapedius muscle For each of the following descriptions, select the MOST RELEVANT structure from the list of options above. Each option may be used once, more than once, or not at all. Question 14 Carries parasympathetic secretory fibres to the nasal glands Ans: C Explanation / reference to students: GPN carries parasympathetic fibers to the nasal glands through the pterygopalatine ganglion (PPG) and they are distributed to the different branches of the PPG in the nasal cavity. Question 15 Passes medial to the malleus Ans: A Explanation / reference to students: Chorda tympani (CT) is the only structure passing behind the malleus. CT arises at the posterior wall of the middle ear from the main branch of the facial nerve before it exits the stylomastoid foramen. From the facial nerve, CT courses anteriorly and passes behind the malleus and exits the anterior wall through the petrotympanic fissure. Question 16 Is innervated by the mandibular nerve Ans: H Explanation / reference to students: Tensor tympani is supplied by CNV3. This is one of the cranial nerves supplying structures in the middle ear. The other two cranial nerves are CNVII (stapedius muscle) and CNIX (sensory through the tympanic plexus) Question 17 Neutralises the pressure between the middle ear and nasopharynx Ans: D Explanation / reference to students: Auditory tube connects between nasopharynx and middle ear. The auditory tube is located at the anterior wall of the middle ear. Question 18 Is the origin of the stapedius muscle Ans: G Explanation / reference to students: Pyramid is the origin of stapedius muscle. It’s a bony projection located at the posterior wall of middle ear. [2023 M27 2nd Summative Q42] Otoscopy image of the ear. Standring S. (2016). Gray's Anatomy: The Anatomical Basis of Clinical Practice (41"' ed.). Elsevier Limited. What is the function of the arrowed structure? A. Transmit parasympathetic fibres to otic ganglion B. Transmit parasympathetic fibres to pterygopalatine ganglion C. Transmit taste fibres from the lingual nerve D. Transmit vibrations from the tympanic membrane Ans: C ________________ [2024 M28 Sample Questions] STRUCTURES RELATED TO THE MIDDLE EAR A. Chorda tympani B. Fenestra cochlea C. Greater petrosal nerve D. Pharyngotympanic tube E. Processus cochleariformis F. Promontory G. Pyramid H. Tensor tympani I. Tympanic canaliculus J. Stapedius muscle For each of the following descriptions, select the MOST RELEVANT structure from the list of options above. Each option may be used once, more than once, or not at all. Carries parasympathetic secretory fibres to the nasal glands Ans: C Greater petrosal nerve Explanation / reference to students: GPN carries parasympathetic fibres to the nasal glands through the pterygopalatine ganglion (PPG) and they are distributed to the different branches of the PPG in the nasal cavity. Passes medial to the malleus Ans: A Chorda tympani Explanation / reference to students: Chorda tympani (CT) is the only structure passing behind the malleus. CT arises at the posterior wall of the middle ear from the main branch of the facial nerve before it exits the stylomastoid foramen. From the facial nerve, CT courses anteriorly and passes behind the malleus and exits the anterior wall through the petrotympanic fissure. Is innervated by the mandibular nerve Ans: H Tensor tympani Explanation / reference to students: Tensor tympani is supplied by CNV3. This is one of the cranial nerves supplying structures in the middle ear. The other two cranial nerves are CNVII (stapedius muscle) and CNIX (sensory through the tympanic plexus) ________________ Physiology ________________ HNS01 Overview of the central nervous system / ________________ HNS12 Somaesthetic pathway and somatic sensation [2020 M24 2nd Summative Q52] Meissner corpuscles and Merkel cells are mechanoreceptors on the skin that are important for detecting motion and texture of an object, respectively. Which property is MOST DIFFERENT between the two mechanoreceptors and BEST explains their functional specificity? A. Conduction velocity of action potential B. Localisation on the skin C. Rate of adaptation D. Size of receptive field Ans: C [2022 M26 2nd Summative EMQ Q3] III. SOMATOSENSORY SYSTEM TRANSMITS TACTILE AND PAIN INFORMATION FROM EXTERNAL AND INTERNAL LOCATIONS ON THE BODY AND HEAD A. Caudate nucleus B. Cuneate nucleus C. Gracile nucleus D. Lateral lemniscus E. Medial lemniscus F. Periaqueductal grey G. Red nucleus H. Spinothalamic tracts I. Ventral posterolateral nucleus of thalamus J. Ventral posteromedial nucleus of thalamus For each of the following losses of sensations and function, select the location of the causative lesion from the list of options above. Each option may be used once, more than once, or not at all. Q10 Loss of proprioception of the lower and upper body Ans: E Q11 Loss of pain modulation Ans: F Q12 Loss of upper body two-point discriminative touch sensation Ans: B Q13 Loss of facial sensation Ans: J [2023 M27 2nd Summative Q53] Somatosensory evoked potentials are performed by applying electrical stimuli to the wrist while recording from the cortical region. Which neural structure represents the conduction time at P20? A. Brachial plexus B. Median nerve C. Occipital lobe D. Somatosensory cortex Ans: D Ref: Physiology Practical ________________ HNS13 Pain pathway and mechanisms of pain [2020 M24 2nd Summative Q41] Pain sensation can be modulated by different parts of the central nervous system. Which region of the brainstem integrates pain signals? A. Periaqueductal gray B. Pontine nucleus C. Red nucleus D. Substantia nigra Ans: A [2021 M25 2nd Summative Q53] Patients with myocardial infarction may manifest pain originating from the left arm. What feature of nociceptors in the arm and the heart can BEST explain the referred pain phenomenon by the patients? A. They connect to the same second order neuron in spinal cord. B. They have different rates of action potential conduction. C. They have different sizes of receptive fields. D. They project to different areas of the brain. Ans: A ________________ HNS15 Motor system [2019 M23 2nd Summative Q50] A 78-year-old gentleman had a history of diabetes mellitus and hypertension. He developed a lacunar infarct involving the right basal ganglia region as a result of obstruction of the deep penetrating arteries by lipohyalinosis. Which of the following signs and symptoms is MOST LIKELY to be seen in this patient? A. Sudden onset of fixed and dilated pupil on the right side B. Sudden onset of headache and projectile vomiting C. Sudden onset of left hemiplegia D. Sudden onset of loss of consciousness E. Sudden onset of right hemiplegia Ans: C [2019 M23 2nd Summative Q62] The cerebral arteries supply specific territories in the brain. An elderly man suddenly developed right lower limb weakness due to complete occlusion of one of the arteries. The right upper limb and left side of the body were not affected. Which of the following arteries is the MOST LIKELY culprit? A. Left anterior cerebral artery B. Left middle cerebral artery C. Left posterior cerebral artery D. Right posterior cerebral artery E. Right vertebral artery Ans: A [2019 M23 2nd Summative Q63] A 40-year-old gentleman presented with a unilateral facial nerve palsy involving the lower face but not the upper face. His other cranial nerves were normal. There was no limb weakness. Where is the MOST LIKELY site of disease? A. Caudate nucleus B. Frontal lobe cortex C. Medulla oblongata D. Parietal lobe cortex E. Parotid gland Ans: B Comment from Question Writer * The patient had an “upper motor neurone” type of facial nerve palsy – only the lower but not the upper face was affected. This contrasts with a “lower motor neurone” type of facial nerve palsy in which both upper and lower faces would be affected. This has been explicitly taught during WCS as “the kind of question that will likely come up”. * Option A – this is incorrect because a lesion here does not usually cause isolated cranial nerve palsy. * Option B – this is correct answer as the motor cortex (i.e. “upper motor neurone”) is involved. * Option C – this is incorrect because the facial nerve leaves the brainstem well above the medulla oblongata. * Option D – this is incorrect because the parietal cortex is mainly sensory in function. * Option E – this is incorrect because the facial nerve within the parotid gland is a “lower motor neurone”. [2020 M24 2nd Summative Q33] The patient's tongue points to the left when protruding. Which nerve is MOST LIKELY damaged? A. Left CN X B. Left CN XII C. Right CN IX D. Right CN XII Ans: B [2022 M26 2nd Summative Q63] A 56-year-old man presents with a sudden onset of right upper and lower face weakness. His speech is normal. There is no limb weakness. Where is the MOST LIKELY site of disease? A. Inferior frontal gyrus B. Internal acoustic meatus C. Internal capsule D. Medulla oblongata Ans: B [2024 M28 Sample Questions] The middle cerebral artery (MCA) is a major source of blood supply to the brain. Which of the following would MOST LIKELY be present following an acute right MCA occlusion? A. Expressive dysphasia B. Impaired conjugate eye movement C. Lower facial weakness D. Lower limb muscle fasciculation Ans: C Explanation / reference to students: Option A: Unlikely since it is right MCA ischaemia; the Broca’s area is mostly on the left. Option B: Frontal eye field is not likely to be affected. Option C: Correct - a cardinal sign of upper motor neuron lesion due to MCA occlusion. Option D: A sign of lower motor neuron lesion. ________________ HNS16 Cerebellum [2019 M23 2nd Summative Q58] The vestibulo-ocular reflex stabilises gaze for clear vision during head movement. Which brain region in the diagram provides inhibitory input to the neural circuit mediating this reflex? A. Cerebellar nodulus B. Midbrain C. Pontine reticular formation D. Tectum E. Tegmentum Ans: A [2020 M24 2nd Summative Q40] Inferior olivary nucleus conveys motor information to Purkinje cells. Which peduncles mediate the connection between inferior olivary nucleus to Purkinje cells? A. Cerebral peduncles B. Inferior cerebellar peduncles C. Middle cerebellar peduncles D. Superior cerebellar peduncles Ans: B [2022 M26 2nd Summative Q59] Cerebellum is one of the three main neural areas for voluntary motor control. What is the test for identifying cerebellar dysfunction? A. Plantar response B. Reaction time C. Spiral analysis D. Tandem walking test Ans: D ________________ [2023 M27 2nd Summative Q52] Basal ganglia, cerebellum and cerebral cortex are the three main neural areas for motor control. Which clinical test specifically identifies cerebellar dysfunction? A. Neuropathy test B. Plantar reflex test C. Romberg test with eyes open D. Tandem walking test Ans: D ________________ HNS17 Control of movement [2020 M24 2nd Summative Q53] There are multiple descending pathways from the motor cortex and brainstem for the control of posture and movement. What is the major function of the vestibulospinal tract originating from the brainstem? A. Controls axial muscles and posture B. Controls movement of fingers on contralateral side of the body C. Provides input to basal ganglia for planning complex movements D. Provides input to cerebellum for feedback control of movement Ans: A [2020 M24 2nd Summative Q57] The vestibulospinal reflex coordinates head and body movements in maintaining an upright posture. Which brain region is involved in executing this reflex? A. Cerebellar dentate nucleus B. Medullary pyramids C. Medullary reticular formation D. Red nucleus Ans: C Comment from Question Writer * Students might have confusion with respect to postural control versus motor coordination. (C) is the correct answer. * A) Cerebellar dentate nucleus is involved in modifying voluntary movements but NOT for posture control. * B) Medullary pyramids carry the pyramidal tracts originating from the motor cortex and are involved in voluntary movements. * C) Medullary reticular formation is closely linked with the vestibular nucleus to exert descending influences on vestibulospinal reflex. * D) Red nucleus is a midbrain structure for motor coordination but is not related to the vestibulospinal reflex. ________________ HNS26 Salivary and nasal secretion and their effects on taste and smell [2020 M24 2nd Summative Q51] Our body produces around 1.5 litres of saliva every day and it plays a vital role in lubricating food, digestion, and protecting the oral cavity. Which transporter is only involved in the formation of primary saliva? A. Epithelial Na channels (ENaC) B. Na-K-Cl cotransporter (NKCC) C. Na/HC03 cotransporter D. Na/K.-ATPase Ans: B Explanation / reference to students: Only NKCC is involved in the formation of primary saliva. [2021 M25 2nd Summative Q52] The salivary glands are exocrine glands that produce saliva through a system of ducts. Which of the following glands are related to the Wharton duct? A. Infraorbital glands B. Parotid glands C. Sublingual glands D. Submandibular glands Ans: D Comment from Question Writer * My submitted MCQ is a very simple, easy, and straightforward one without the need of special analysis of any knowledge they acquired from the class. It is simply recalling memory. * My only suggestion to the students is to revise the teaching material thoroughly. [2022 M26 2nd Summative Q58] The salivary glands are controlled by the autonomic nervous system. Damage to which ganglia can disturb the saliva production from parotid glands? A. Gasserian B. Geniculate C. Otic D. Pterygopalatine Ans: C ________________ [2024 M28 Sample Questions] Our body produces around 1.5 liters of saliva every day and it plays a vital role in lubricating food, digestion, and protecting the oral cavity. Which transporter is only involved in the formation of primary saliva? A. Epithelial Na channels (ENaC) B. Na-K-Cl cotransporter (NKCC) C. Na/HCO3 cotransporter D. Na/K-ATPase Ans: B Explanation / reference to students: Only NKCC is involved in the formation of primary saliva. ________________ HNS30 Swallowing and speech / ________________ HNS38 Vision [2019 M23 2nd Summative Q61] Depth perception is the visual ability to perceive the distance of an object. Both monocular cues and binocular cues can provide depth information to our nervous system. Apart from binocular parallax, which depth cue can only be detected using both eyes? A. Interposition B. Linear perspective C. Ocular convergence D. Relative size E. Texture gradient Ans: C [2023 M27 2nd Summative Q96] Visual features are extracted and interpreted at specific parts of the brain. Where is spatial and form recognition being perceived along the visual pathway? A. Higher visual cortex B. Lateral geniculate nucleus C. Optic radiation D. Parietal lobe Ans: A ________________ HNS39 Eye movements [2019 M23 2nd Summative Q56] The amount of light reaching the retina is regulated by the pupillary light reflex. Which process mediates the pupillary response? A. Activation of neurons in the oculomotor nucleus B. Activation of neurons in the superior colliculus C. Contraction of the ciliary muscle D. Contraction of the iris sphincter muscle E. Descending activity of the frontal eye field Ans: D A: Incorrect, activation of neurons in the Edinger-Westphal nucleus (HNS04) [2020 M24 2nd Summative Q54] Eye movements are controlled by muscles innervated by the cranial nerves. What clinical observation is a characteristic of a lesion of cranial nerve III? A. Deviation of eye laterally and downwards B. Deviation of eye laterally and upwards C. Deviation of eye medially and downwards D. Deviation of eye medially and upwards Ans: A [2020 M24 2nd Summative Q56] Pupillary light reflex regulates the intensity of light that reaches the retina. Pupillary response is controlled by which of the following? A. Pupil constriction is due to activation of neurons in the interstitial nucleus of Cajal. B. Pupil constriction is due to relaxation of sphincter muscle. C. Pupil dilatation is regulated by neurons in the Edinger-Westphal nucleus. D. Pupil dilatation is regulated by sympathetic nerve fibres. Ans: D [2021 M25 2nd Summative Q54] Eye movements are controlled by muscles innervated by the cranial nerves. What clinical observation is characterised by a lesion of the cranial nerve IV? A. Deviation of eye medially and downwards B. Deviation of eye medially and upwards C. Deviation of the eye outwards and downwards D. Deviation of the eye outwards and upwards Ans: D ________________ [2024 M28 Sample Questions] Eye movements are controlled by muscles innervated by the cranial nerves. What clinical observation is a characteristic of a lesion of cranial nerve III? A. Deviation of eye laterally and downwards B. Deviation of eye laterally and upwards C. Deviation of eye medially and downwards D. Deviation of eye medially and upwards Ans: A Explanation / reference to students: Students are advised to pay attention on eye movement during specific nerve lesion. [2024 M28 Sample Questions] Eye movements are controlled by muscles innervated by the cranial nerves. What clinical observation is characterized by a lesion of cranial nerve IV? A. Deviation of eye medially and downwards B. Deviation of eye medially and upwards C. Deviation of the eye outwards and downwards D. Deviation of the eye outwards and upwards Ans: D Explanation / reference to students: Students are advised to pay attention to eye movement during specific nerve lesions. ________________ HNS41 Audition [2021 M25 2nd Summative Q51] Acoustic startle reflex is a defensive response triggered by a sudden loud noise of > 80 dB. Which neural area plays an important role in generating the startle response? A. Inferior colliculus B. Inferior olive C. Lateral lemniscus nucleus D. Trapezoid body Ans: A ________________ HNS42 Vestibular system [2019 M23 2nd Summative Q57] Vestibular apparatus in the inner ear senses head movements. What is the stimulus that excites sensory hair cells underneath the otoconia crystals? A. Endolymph motion B. Optokinetic input C. Perilymph motion D. Rotational motion E. Translational motion Ans: E Comment from Question Writer * This information was covered during the lecture on “Vestibular System”. * In addition, a diagram on “otoconia crystals” and their association with “translational head movement” was in a ppt slide #10 that I uploaded onto the Faculty website before the lecture. * Furthermore, the association of “endolymph motion” with “cupula” of the semicircular canal was in another ppt slide #7. * Therefore, students who chose Option A (“endolymph motion”) probably did not recognise such detailed information provided to them. ________________ HNS43 Higher cortical function [2020 M24 2nd Summative Q39] To test the language ability of a patient, a neurologist asks the patient to describe the picture. Which brain region is involved in the comprehension of language? A. Amygdala B. Broca area C. Putarnen D. Wernicke area Ans: D Explanation / reference to students: This is a question for the neuroanatomy of language. Students are advised to tell the difference between the region for making language and comprehension of language. Options A and C are just other anatomical regions for negative emotion and motor. [2021 M25 2nd Summative Q40] A 55-year-old man had trouble reading the newspaper. The visual field examination showed a right homonymous inferior quadrantic hemianopia/quadrantanopia. Lesion at which location would MOST LIKELY correlate with his visual problem? A. Left Meyer's loop B. Left superior bank of primary visual cortex C. Right lateral geniculate nucleus D. Right parietal optic radiation Ans: B Explanation / reference to students: If visual field is divided into 4 quadrants, each quadrant projects to different part of the retina in an up-side-down, left-right reversed manner. The signals from different part of retina will go through different fiber tracts and project to different parts of lateral geniculate nucleus and eventually different parts of the primary visual cortex. Understanding the visual pathway is important for diagnosis of visual field loss and the potential lesion site. ________________ [2022 M26 2nd Summative Q45] The picture below shows the Brodmann areas of the cortex: (Source: netterimages.com; Image ID: 29279) What is the cortical region of Brodmann areas numbered 44, 45 and 47 (indicated by arrows)? A. Broca's area B. Hippocampus C. Olfactory cortex D. Wernicke's area Ans: A Explanation / reference to students: This is a question to test the neuroanatomy of language. Although students have not been taught with Brodmann number, they should learn the area for Broca’s area. [2022 M26 2nd Summative Q57] Thalamocortical projections play an important role in cortical functions. A lesion of which thalamic nucleus would lead to personality changes and apathy? A. Medial geniculate nucleus B. Mediodorsal nucleus C. Ventral lateral nucleus D. Ventral posterolateral nucleus Ans: B Explanation / reference to students: Different nuclei in thalamus receive different inputs and have different thalamocortical projections. The mediodorsal nucleus of thalamus relays inputs from the amygdala, olfactory cortex, and other cortical regions, and projects to the prefrontal cortex and the limbic system. It plays important roles in cognition, executive function, memory, personality, and emotion. A: The medial geniculate nucleus relays auditory information and projects to the primary auditory cortex. B:The ventral anterior nucleus receives neuronal inputs from the basal ganglia and projects to the premotor cortex. D: The ventral posterolateral nucleus receives information from the spinothalamic tract and the dorsal column medial lemniscus tract, and projects this sensory information to the primary somatosensory cortex. [2023 M27 2nd Summative Q43] A patient developed a tumour at region X. (Source: Anatomy dissection lab; HKU) What visual disturbances are MOST LIKELY to be seen in the patient? A. Binasal hemianopsia B. Bitemporal hemianopsia C. Left homonymous hemianopsia D. Right homonymous hemianopsia Ans: B [2023 M27 2nd Summative Q54] A 55-year-old man had a brain tumour located in the Meyer's loop (temporal optic radiation) in the left hemisphere and he had partial vision loss. What kind of visual field defect did he suffer from? A. Left lower quadrant homonymous hemianopia B. Left upper quadrant homonymous hemianopia C. Right lower quadrant homonymous hemianopia D. Right upper quadrant homonymous hemianopia Ans: D [2024 M28 Sample Questions] To test the language ability of a patient, a neurologist may give instructions to the patient. Which brain region is involved in language comprehension? A. Amygdala B. Broca's area C. Putamen D. Wernicke's area Ans: D Explanation / reference to students: This is a question for the neuroanatomy of language. Students are advised to tell the difference between the region for making language and comprehension of language. Options A and C are just other anatomical regions for negative emotion and motor. [2024 M28 Sample Questions] The picture below shows the Brodmann areas of the cortex: (Source: netterimages.com; Image ID: 29279) What is the cortical region of Brodmann areas numbered 44, 45 and 47 (indicated by arrows)? A. Broca's area B. Hippocampus C. Olfactory cortex D. Wernicke's area Ans: A Explanation / reference to students: This is a question to test the neuroanatomy of language. Although students have not been taught with Brodmann number, they should learn the area for Broca’s area. ________________ [2024 M28 Sample Questions] A 55-year-old man had trouble reading the newspaper. The visual field examination showed a right homonymous inferior quadrantic hemianopia/quadrantanopia. Lesion at which location would MOST LIKELY correlate with his visual problem? A. Left Meyer's loop B. Left superior bank of primary visual cortex C. Right lateral geniculate nucleus D. Right parietal optic radiation Ans: B Explanation / reference to students: If visual field is divided into 4 quadrants, each quadrant projects to different part of the retina in an up-side-down, left-right reversed manner. The signals from different part of retina will go through different fiber tracts and project to different parts of lateral geniculate nucleus and eventually different parts of the primary visual cortex. Understanding the visual pathway is important for diagnosis of visual field loss and the potential lesion site. [2024 M28 Sample Questions] Thalamocortical projections play an important role in cortical functions. A lesion of which thalamic nucleus would lead to personality changes and apathy? A. Medial geniculate nucleus B. Mediodorsal nucleus C. Ventral lateral nucleus D. Ventral posterolateral nucleus Ans: B Explanation / reference to students: Different nuclei in thalamus receive different inputs and have different thalamocortical projections. The mediodorsal nucleus of thalamus relays inputs from the amygdala, olfactory cortex, and other cortical regions, and projects to the prefrontal cortex and the limbic system. It plays important roles in cognition, executive function, memory, personality, and emotion. A: The medial geniculate nucleus relays auditory information and projects to the primary auditory cortex. B:The ventral anterior nucleus receives neuronal inputs from the basal ganglia and projects to the premotor cortex. D: The ventral posterolateral nucleus receives information from the spinothalamic tract and the dorsal column medial lemniscus tract, and projects this sensory information to the primary somatosensory cortex. ________________ HNS46 Limbic system, learning and memory [2019 M23 2nd Summative Q42] Which brain region mediates drug addictive behaviour? A. Amygdala B. Hippocampus C. Nucleus accumbens D. Prefrontal cortex E. Ventral tegmental area Ans: E [2024 M28 Sample Questions] A patient has dementia from Alzheimer disease. Which brain region is MOST LIKELY to show early neuronal loss? A. Entorhinal cortex B. Insula C. Pineal gland D. Putamen Ans: A Explanation / reference to students: This is a question for neuroanatomy of neurodegeneration in Alzheimer disease. Prior to neuronal loss in the hippocampus, the most affected region is the entorhinal cortex. ________________ HNS47 Behavioral neuroscience [2019 M23 2nd Summative Q60] Activity of which brain centre responsible for a depressed mood? A. Dorsolateral prefrontal cortex B. Lateral entorhinal cortex C. Parahippocampal gyrus D. Subgenual anterior cingulate cortex E. Ventromedial nucleus of the hypothalamus Ans: D Out of syllabus Comment from Question Writer * This question is from the lecture on “Behavioral Neuroscience”. The most relevant slide is in the section on “Emotions”, with the heading “Brain centers for emotions”. The text of this slide is as follows: * “Emotions and mood are controlled by the complex interactions between prefrontal cortex (ventromedial prefrontal and anterior cingulate) and subcortical nuclei (amygdala and ventral striatum). This system is regulated by input from dorsolateral prefrontal cortex, mediating the interaction between cognitions (perceptions and thoughts) and emotions.” * Thus, the correct answer anterior cingulate cortex is listed as a brain centre that controls emotions and mood. Another answer, dorsolateral prefrontal cortex is also mentioned, but the text indicates that it regulate the “emotion centres” through exerting “cognitive control”. * One possible reason for the low rate of correct responses for this question is that the word “depressed” does not appear on the slide, so that some students may not have linked the slide to the question, which is about “depressed mood”. * However, this slide is in the section on “Emotions”, and depression is a main topic in this section, with the slides: * “Why emotions evolved”; “Anxiety”; “Phobias”; “Depression”; “Measures of depressed mood”; “Arousal and monoamines”; “Brain centres for emotions”; “Negative cognitive bias” * Another issue may be that the slide does not qualify “anterior cingulate” by “subgenual”, as in answer D. Some student may not have appreciated that subgenual anterior cingulate is part of anterior cingulate. * The final slide of the lecture is a summary slide with the heading “Key Points”. The text of the slide are as follows: * Theories of motivation (instincts, drives and incentives), involving dopamine as the neurotransmitter for incentive salience, are particularly relevant for understanding eating disorders * Classical and operant conditioning provide explanations for simple learned behaviors and a model for drug addiction. They form the basis of behavior therapy for fears and phobias (exposure) * Anxiety and depression are abnormal mood states caused by dysfunction of neurotransmitter systems (serotonin and noradrenaline) and brain centers (amygdala, anterior cingulate cortex). * Dopamine hyper-reactivity leads to psychotic symptoms through inappropriate incentive salience and impaired sensory gating * From the third key point, the students should have linked depression and abnormal mood states to anterior cingulate cortex. * In summary, there are some minor discrepancies between the lecture and this question in terms of the wording of the key concepts, which may have caused confusion to some students. [2021 M25 2nd Summative Q55] Incentive salience is thought to be signalled by which of the following neurotransmitter systems? A. Acetylcholine B. Dopamine C. Noradrenaline D. Serotonin Ans: B [2022 M26 2nd Summative Q60] A student presents with the conviction of being followed by random strangers. Dysregulation of which neurotransmitter pathway is the MOST LIKELY cause? A. Basal forebrain cholinergic projection to the hippocampus B. Brainstem noradrenergic projection to the prefrontal cortex C. Brainstem serotonergic projection to the cingulate gyrus D. Midbrain dopamine projection to the striatum Ans: D [2023 M27 2nd Summative Q55] A young woman has recently developed compulsive use of an addictive substance. Dopamine release in which brain region is MOST RESPONSIBLE for driving her compulsive substance use? A. Dorsal striatum B. Hippocampus C. Hypothalamus D. Ventral striatum Ans: A Comment from Question Writer The relevant material is in the lecture on “Behavioral Neuroscience”, Slides 28 – 30. The bullet points in these slides are: * Some drugs can directly stimulate the reward system, even though they are not helpful for survival or reproduction. * Drug-taking behaviour is reinforced by this property, resulting in escalating drug use with serious negative consequences. * Drugs that increase dopamine release initially activate the ventral striatum, activating the reward circuit and reinforcing further drug use * As drug use becomes more frequent, external stimuli associated with drug use also increase dopamine release, and elicit craving for the drug * Drug use becomes less pleasurable and more compulsive, the behavior being driven by dopamine release in the dorsal striatum. These bullet points describe how the initial stages of drug addiction is driven by the ventral striatum, while the later progression to compulsive use is driven by the dorsal striatum. [2024 M28 Sample Questions] A woman has recently become depressed. Which of the following is the MOST LIKELY trigger? A. A forthcoming stressful event B. A lack of external stimulation C. Disturbance of normal routine D. The loss of a significant resource Ans: D Explanation / reference to students: The topic is Behavioural Neuroscience. Depression is usually triggered by the actual or anticipated loss of rewards or the capacity to obtain rewards (i.e. resources). Lack of stimulation causes boredom. Disruption of normal routine and forthcoming stressful event are more likely to trigger anxiety and depression. [2024 M28 Sample Questions] A young man has recently started to use illicit drugs. Which brain area is MOST RESPONSIBLE for the reinforcement of his drug taking behaviour? A. Dorsal striatum B. Periaqueductal grey C. Prefrontal cortex D. Ventral striatum Ans: D Explanation / reference to students: The topic is Behavioural Neuroscience. Drugs that increase dopamine release initially activate the ventral striatum, activating the reward circuit and reinforcing further drug use. As drug use becomes more frequent, external stimuli associated with drug use also increase dopamine release, and elicit craving for the drug. Eventually drug use becomes less pleasurable and more compulsive, the behaviour being driven by dopamine release in the dorsal striatum. ________________ Pathology ________________ HNS11 Pathology of CNS infection [2019 M23 2nd Summative Q46] A 50-year-old poorly controlled diabetic patient presented with headache, night sweating, fever and weight loss for 3 months. His cerebrospinal fluid (CSF) also showed high protein, high sugar and lymphocytic pleocytosis. Bacterial, mycobacterial and fungal cultures were negative. His symptoms fully responded to a combination of anti-tuberculous drug treatment. Why were microbiological cultures for Mycobacterium tuberculosis negative in this patient? A. Mycobacterium tuberculosis are often walled off by granulomatous reaction B. Mycobacterium tuberculosis cannot spread through the subarachnoid space C. Mycobacterium tuberculosis is partially suppressed by antibiotic from the family doctor D. Obstruction of CSF circulation in the subarachnoid space from brain to spinal cord E. The high sugar level in CSF suppressed the growth of Mycobacterium tuberculosis Ans: A [2019 M23 2nd Summative Q47] A 55-year-old man presented with a 16-month history of progressive loss of memory and cognitive ability, personality changes, unsteadiness and clumsiness and more recently myoclonic jerks. He has previously been well and has no known immunocompromised conditions. The cerebrospinal fluid examination revealed no increase of white cells or total protein. Which of the following aetiological agents would you consider to be the MOST LIKELY cause of his illness? A. Cryptococcus B. Cytomegalovirus C. Herpes simplex type 1 D. John Cunningham (JC) virus E. Prions Ans: E [2020 M24 2nd Summative Q47] Which of the following pathological changes and corresponding consequences is MOST COMMONLY observed in tuberculous meningitis? A. Attack of myelin sheath by body immune system and demyelination B. Fibrosis with obliteration of subarachnoid space and hydrocephalus C. Lipohyalinosis of penetrating arteries and infarction of basal ganglia D. Neuronophagia (engulfment of neurons by microglia) and loss of anterior horn cells Ans: B ________________ [2021 M25 2nd Summative Q49] Fungi can cause infection of the central nervous system. However, most fungal infections are opportunistic, affecting patients with underlying impaired cellular immunity. Which of the following fungi is MOST LIKELY to cause fungal meningitis in an otherwise healthy individual? A. Aspergillus B. Candida C. Cryptococcus D. Mucormycosis Ans: C Explanation / reference to students: The topic is CNS infection. Cryptococcus can affect healthy people without underlying disease. Options A, B and D – these fungi tend to cause disease only in immunocompromised patient and thus are known as opportunistic infections. [2023 M27 2nd Summative Q49] Fungi can cause infection of the central nervous system. However, most fungal infections are opportunistic, affecting patients with underlying impaired cellular immunity. Which of the following fungi is MOST LIKELY to cause fungal meningitis in an otherwise healthy individual? A. Aspergillus B. Candida C. Cryptococcus D. Mucormycosis Ans: C Explanation / reference to students: The topic is CNS infection. Cryptococcus can affect healthy people without underlying disease. Options A, B and D – these fungi tend to cause disease only in immunocompromised patient and thus are known as opportunistic infections. [2024 M28 Sample Questions] opportunistic, affecting patients with underlying impaired cellular immunity. Which of the following fungi is MOST LIKELY to cause fungal meningitis in an otherwise healthy individual? A. Aspergillus B. Candida C. Cryptococcus D. Mucormycosis Ans: C Explanation / reference to students: The topic is CNS infection. Cryptococcus can affect healthy people without underlying disease. Options A, B and D – these fungi tend to cause disease only in immunocompromised patient and thus are known as opportunistic infections. [2024 M28 Sample Questions] Different types of pathological damage can lead to different modes of cell injury and pathological reaction. Which of the following is the characteristic pathological change of an acute cerebral infarction of 1 to 2-week-old? A. Formation of granuloma with caseous necrosis at the centre bordered by epithelioid cells B. Healing by fibrosis with progressive transformation into a fibrous scar C. Infiltration of lymphocytes and microglia that attack and engulf the neurons D. Infiltration of macrophages with abundant foamy cytoplasm Ans: B Explanation / reference to students: The topic is: Cerebral infarction. Different types of insult lead to different defence and healing pattern, which has implication on the associated disease course and complications. Cerebral infarction is a type of ischaemic necrosis which in the brain is characterised by liquefactive necrosis because of the high lipid content. After ischaemic necrosis of the neurons and glial cells, the body scavenger system gets activated to remove the cell debris, thus the macrophages having engulf the necrotic cell debris with high lipid content, contain abundant foamy cytoplasm. Option A – caseous necrosis is the defence pattern against mycobacteria. Option B – there are no fibroblasts within the brain so fibrous scar is rarely formed. Option C – Lymphocytes and microglia will attach the neurons only upon immune activation, commonly as a consequence of viral infection with the neurons expressing viral proteins, thus being recognized by the immune system. ________________ HNS18 Pathology of raised intracranial pressure and cerebrovascular disease [2019 M23 2nd Summative Q53] A 35-year-old man was brought into the Accident and Emergency Department after a car accident and sustained a skull fracture. He complained of right homonymous hemianopia and rapidly became comatose. An urgent computed tomography scan showed a large epidural haematoma on the left side of his brain. What is the MOST LIKELY cause of his visual problem? A. Compression of the aqueduct of Sylvius causing obstructive hydrocephalus B. Compression of the left occipital lobe C. Compression of the left oculomotor nerve D. Compression of the left posterior cerebral artery E. Compression of the optic nerve Ans: D Comment from Question Writer * It is difficult as some logics on the lesion causing transtentorial herniation to compress posterior cerebral artery is needed. [2020 M24 2nd Summative Q48] A 76-year-old gentleman had long standing hypertension but often forgot to take his drug. One day he was found unconscious on the street and brought into the Accident and Emergency Department. An urgent computed tomography scan of the brain showed the presence of a 1x1x0.5 cm haematoma in his pons. He died shortly afterwards. Which of the following pathological changes is MOST LIKELY associated with the cause of his haematoma in the pons? A. Berry (saccular) aneurysm in the posterior cerebral artery B. Deposition of amyloid in the cerebral blood vessels C. Emboli from the heart obstructing the pontine arteries D. Lipohyalinosis with microaneurysm in the deep penetrating pontine arteries Ans: D ________________ [2023 M27 2nd Summative EMQ Q3] III. RAISED INTRACRANIAL PRESSURE CAUSED BY SPACE OCCUPYING LESION A. Aqueduct of Sylvius B. Fourth ventricle C. Left middle cerebral peduncle D. Left motor cortex E. Left posterior cerebral artery F. Midbrain G. Right middle cerebral peduncle H. Right oculomotor nerve I. Right optic nerve J. Right posterior cerebral artery A 34-year-old taxi driver had a road traffic accident and sustained head injury with skull fracture. An urgent computed tomography (CT) scan upon admission to the Accident and Emergency Department showed a large epidural haematoma beneath the right temporal skull fracture, which compressed directly on the underlying right cerebral hemisphere including the right motor and sensory cortex, and resulting in midline shift to the left side. His Glasgow coma scale deteriorated rapidly with onset of the following neurological signs. For each of the following neurological signs, select the structure in the brain from the list of options above which is MOST LIKELY to be compressed/damaged because of the space occupying lesion in the right cerebral hemisphere. Each option may be used once, more than once, or not at all. Question 6 Right hemiplegia Ans: C Question 7 Progressive deterioration in consciousness Ans: F Question 8 Papilloedema Ans: I ________________ [2024 M28 Sample Questions] A previously healthy 34-year-old lady developed sudden severe headache and was admitted into hospital. There was no history of hypertension or prior head injury. A computed tomography scan of the brain was performed, and showed a thin film of blood in the subarachnoid space. The brain itself showed no additional abnormal bleeding site. What is the MOST LIKELY underlying pathology and their anatomical location? A. Berry (saccular) aneurysm located at the bifurcation of the internal carotid artery into the middle cerebral and posterior communicating artery. B. Berry (saccular) aneurysm located at the middle segment of the basilar artery. C. Microaneurysm located at the deep penetrating arteries supplying the basal ganglia. D. Microaneurysm located at the pontine arteries. Ans: A Explanation / reference to students: The topic is Cerebrovascular disease. The sudden onset of severe headache could be caused by subarachnoid haemorrhage with the blood irritating the meninges could also lead to neck rigidity. The most common cause of sudden subarachnoid haemorrhage in an otherwise healthy individual is due to berry (saccular) aneurysm that has ruptured. Why are the other options wrong? Berry aneurysm is different from the microaneurysm in penetrating arteries due to chronic hypertension. Berry aneurysm is not related to hypertension and it tend to be located in bifurcation of major cerebral arteries in Circle of Willis, located in the subarachnoid space. Whereas microaneurysm of hypertension tends to locate in deep penetrating arteries within brain substance (especially basal ganglia, pons and cerebellum) and if ruptured, give risk to intracerebral haemorrhage. ________________ HNS32 Pathology of intracranial tumours / ________________ HNS33 Common ENT cancers: Anatomic and physiologic appraisal / ________________ HNS34 Common ENT inflammatory diseases: Anatomic and physiologic appraisal [2020 M24 2nd Summative Q60] A 5-year-old boy has presented with ear pain and fever for 2 days. He also suffers from nasal obstruction and thick nasal discharge. On examination, the tympanic membrane is very congested and bulging. Which of the following is the clinical diagnosis? A. Acute otitis media B. Cholesteatoma C. Chronic suppurative otitis media D. Otitis extema Ans: A [2023 M27 2nd Summative Q57] A 50-year-old woman presents with episodic right otorrhoea for 1 year. Her hearing on the right side is also impaired. On examination, there is central perforation of the right tympanic membrane and congestion of the middle ear mucosa. Which of the following is the clinical diagnosis? A. Cholesteatoma B. Chronic suppurative otitis media C. Labyrinthitis D. Otomycosis Ans: B [2024 M28 Sample Questions] A 4-year-old girl presented with right ear pain and fever for 2 days. She also suffers from nasal obstruction and thick nasal discharge. On examination, the tympanic membrane is very congested and bulging. Which of the following is the clinical diagnosis? A. Acute otitis media B. Chronic suppurative otitis media C. Furunculosis D. Otomycosis Ans: A Explanation / reference to students: The features on the tympanic membrane are most important. Chronic suppurative otitis media is incorrect since the tympanic membrane would be perforated in this disease. Furunculosis is incorrect since this localized external ear infection would obscure the view of the tympanic membrane. Otomycosis is incorrect since there is no mention of fungal spores on examination. ________________ HNS35 NPC and other head and neck tumours [2019 M23 2nd Summative Q52] A 35-year-old lady presented with an enlargement of the left parotid gland. Fine needle aspiration showed cytologically benign spindle cells in a myxoid stroma together with small duct-like spaces. What is the MOST LIKELY diagnosis? A. Adenoid cystic carcinoma B. Mucoepidermoid carcinoma C. Pleomorphic adenoma D. Sialadenitis E. Warthin's tumour Ans: C [2021 M25 2nd Summative Q48] In the multistep progression to the development of nasopharyngeal carcinoma, which of the following factors is regarded as being important for the progression from early premalignant change of the epithelium to irreversible malignant transformation? A. DNA repair damage B. Epstein-Barr virus infection C. Exposure to salted fish D. Genetic susceptibility Ans: A [2022 M26 2nd Summative Q54] A 25-year-old lady presented with an enlargement of the right parotid gland. Fine needle aspiration showed cytologically benign spindle cells in a myxoid stroma plus small duct like spaces. What is the MOST LIKELY diagnosis? A. Adenoid cystic carcinoma B. Mucoepidermoid carcinoma C. Pleomorphic adenoma D. Sialadenitis Ans: C ________________ Microbiology ________________ HNS05 Infection of the CNS I: Bacterial and fungal infections [2019 M23 2nd Summative Q51] A 7-year-old child developed a sudden onset of high fever and appeared drowsy when brought into the Accident and Emergency Department. The doctor found that the child showed neck rigidity. A lumbar puncture was performed and the cerebrospinal fluid showed a high white cell count with predominantly neutrophils. Which of the following is the MOST LIKELY diagnosis? A. Cysticercosis B. Group B streptococcal meningitis C. Meningococcal meningitis D. Tuberculous meningitis E. Viral meningoencephalitis Ans: C [2020 M24 2nd Summative Q46] A 4-year-old, previously healthy child, started to develop high fever two days ago, rapidly progressed to neck rigidity and decrease in consciousness. A lumbar puncture was performed when he was admitted into the hospital. The cerebrospinal fluid showed a high white blood cell count with predominantly polymorphs. Which of the following is the MOST LIKELY disease? A. Cysticercosis B. Meningococcal meningitis C. Tuberculous meningitis D. Viral meningoencephalitis Ans: B [2020 M24 2nd Summative EMQ Q3] III. MICROBIOLOGICAL AETIOLOGIES OF CENTRAL NERVOUS SYSTEM INFECTIONS A. Candida albicans B. Cryptococcus neoformans C. Enterovirus A71 D. Escherichia coli E. Herpes simplex virus 1 F. Mycobacterium tuberculosis G. Neisseria gonorrhoeae H. Neisseria meningitidis I. Streptococcus pneumoniae J. Streptococcus suis In each of the following clinical scenarios, select the MOST L IKELY causative organism from the list of options above. Each option may be used once, more than once, or not at all. ________________ Question 10 A 56-year-old butcher with a good past health was admitted for fever, headache, neck stiffness, and hearing loss for 3 days. Gram-stained smear of his cerebrospinal fluid showed Gram-positive cocci in chains. Ans: J Question 11 A 46-year-old woman with systemic lupus erythematosus on prednisolone was admitted for fever, headache, and neck stiffness for 4 weeks. Gram-stained smear of her cerebrospinal fluid showed Gram-positive bacilli. Ans: F Question 12 A 7-day-old baby girl, born at full term by normal vaginal delivery, was admitted with fever, convulsion, lethargy and bulging fontanelles. Gram-stained smear of her cerebrospinal fluid showed Gram-negative bacilli. Ans: D Question 13 A 4-year-old girl developed fever and vesicular lesions over hands, feet, and lips. Four days later, she developed myoclonic jerks, vomiting, and ataxia. Her skin lesions and faeces were positive for an RNA virus by reverse transcription-polymerase chain reaction. Ans: C Question 14 A 42-year-old man with HIV infection was admitted for fever, headache, and neck stiffness for 6 weeks. India ink stained-smear of his cerebrospinal fluid showed yeast cells with surrounding halo. Ans: B ________________ [2021 M25 2nd Summative Q45] A 7-year-old girl presented with an acute onset of fever, headache, and neck stiffness for 2 days. Cerebrospinal fluid (CSF) analysis revealed raised white cell count of 1300 cells/mm3 with polymorph predominance, elevated protein level, and low CSF-to-serum glucose level. Gram-stained smear of the CSF showed Gram-negative diplococci. Which of the following is the MOST LIKELY organism causing her acute meningitis? A. Corynebacterium diphtheriae B. Escherichia coli C. Klebsiella pneumoniae D. Neisseria meningitidis Ans: D [2021 M25 2nd Summative Q46] Cryptococcal meningitis is an important cause of chronic meningitis in patient with HIV and other immunosuppressed conditions. What is MOST SENSITIVE way of making such a diagnosis? A. Cryptococcal antigen test of CSF B. Fungal culture of CSF C. Indian ink examination of CSF D. PCR test on CSF Ans: A Comment from Question Writer * Cryptococcus neoformans produce very abundant mucoid capsular material giving the fungal colony on agar plate a very mucoid appearance. * The capsular material, glucuronoxylomannan would be released into CSF and give a strongly positive cryptococcal antigen test even when the number of Cryptococcus is very low in the CSF which may not be detectable by culture or PCR. [2022 M26 2nd Summative Q52] A patient develops severe headache after 1 month of foul left nasal discharge. Computed tomography scan showed a 4 cm left frontal lobe hypodense lesion perilesional oedema with midline shift. What is the MOST SUITABLE treatment regimen for this condition? A. Drainage of brain abscess and paranasal sinusitis while giving intravenous antibiotics which cross blood-brain barrier and cover aerobes and anaerobes B. Drainage of brain abscess and paranasal sinusitis while giving intravenous high dose antibiotics with anti-staphylococcal coverage C. Intravenous anti-staphylococcal antibiotics for brain abscess and paranasal sinusitis D. Intravenous high dose antibiotics with aerobic and anaerobic coverage for cerebritis and paranasal sinusitis Ans: A [2023 M27 2nd Summative Q47] A 40-year-old man presented with headache and unsteadiness for 2 weeks. Cerebrospinal fluid (CSF) analysis showed lymphocytic pleocytosis, elevated protein, and low glucose. Indian ink smear on the CSF was positive. Which of the following is a useful antimicrobial to treat his condition? A. Acyclovir B. Amphotericin B C. Isoniazid D. Meropenem Ans: B [2023 M27 2nd Summative Q48] A 50-year-old man presented with low-grade fever, headache, decreased consciousness, and seizure. Which of the following is NOT a contraindication for immediate lumbar puncture? A. Focal neurological deficit B. Presence of fever C. Presence of seizure D. Severe thrombocytopenia Ans: B [2024 M28 Sample Questions] What is expected at the lumbar puncture and examination of the cerebrospinal fluid in a patient with tuberculous or cryptococcal meningitis? A. A high opening pressure B. A high percentage of neutrophils C. A low concentration of protein D. A normal concentration of glucose Ans: A Explanation / reference to students: Tuberculous and cryptococcal meningitis are well-known to be associated with high opening pressure. Reference is made to the lecture and practical on CNS infections. ________________ HNS06 Infections of the CNS II: Viral and prion infections of the CNS [2020 M24 2nd Summative Q44] A 4-year-old boy attending nursery school developed vesicular lesions on the fingers, feet and ulcers in the mouth of 2 days duration. He had a headache and fever. On examination he had neck stiffness and developed flaccid paralysis of his right arm. What is the MOST LIKELY diagnosis? A. Chickenpox infection B. Coxsackie A virus infection C. Enterovirus 7 1 virus infection D. Polio virus infection Ans: C [2022 M26 2nd Summative Q53] A 5-year-old boy developed acute fever, headache, and neck stiffness. Analyses of the cerebrospinal fluid sample obtained before the use of antibiotics were as follows: White cells count 320/mm³ (polymorph 10%, lymphocyte 90%) Protein 0.75 g/L (Normal: 0.15 - 0.45 g/L) Glucose 3.1 mmol/L (Normal: 2.8 - 3.9 mmol/L) Serum glucose 5.5 mmol/L Gram-stained, Ziehl-Neelsen-stained, and India ink-stained smears: negative Which of the following is the MOST LIKELY causative organism of his condition? A. Echovirus B. Neisseria meningitidis C. Rotavirus D. Streptococcus suis Ans: A ________________ HNS31 Infectious causes of upper airway obstruction [2019 M23 2nd Summative Q48] A 3-month-old boy was brought to the Accident and Emergency Department because of fever and progressive sore throat for 2 days. He was drooling saliva and physical examination showed an inspiratory stridor. Which of the following investigations would you perform immediately? A. Computed tomography of the head and neck region B. Direct laryngoscopy C. Nasopharyngeal aspirate for respiratory viruses D. Throat swab for bacterial culture E. Urine for pneumococcal antigen Ans: B ________________ Pharmacology ________________ HNS10 Infections of the CNS III: Principles of antiviral therapy [2021 M25 2nd Summative Q47] A 30-year-old man presents to the hospital with fever and shortness of breath. Chest x ray shows right lower zone consolidation. Empirical antiviral for influenza virus is given. What is the MOST LIKELY antiviral given to this patient? A. A neuraminidase inhibitor B. A nucleoside analogue C. A reverse transcriptase inhibitor D. An inhibitor of DNA terminase complex Ans: A [2021 M25 2nd Summative Q71] A 30-year-old man was brought to the hospital by his wife for acute confusion. Acute encephalitis was suspected, and the doctor prescribed intravenous acyclovir. What is the mechanism of action of acyclovir? A. Inhibit attachment of the virus to cell surface receptor B. Inhibit the release of the virus from cell surface C. Inhibit viral genome replication D. Inhibit viral protein synthesis Ans: C [2023 M27 2nd Summative Q95] Oral antivirals are now clinically approved for the treatment of COVID-19. What is the mechanism of action for molnupiravir? A. Inhibit protease activity B. Inhibit viral release C. Lethal mutagenesis D. Prevent endocytosis Ans: C Comment from Question Writer This question was taught in the lecture “Infections of the CNS III: Principles of antiviral therapy”. It tests whether the students understand the mechanism of action of antiviral drugs. I’ve specifically chosen a very common drug used for COVID-19, molnupiravir. * Option A: inhibits protease - Paxlovid, another COVID drug * Option B: inhibit viral release - oseltamivir, influenza drug * Option D: None of the currently approved COVID antiviral prevent endocytosis ________________ HNS14 Narcotic analgesics / ________________ HNS25 Sedatives and hypnotics [2020 M24 2nd Summative Q49] John's mother reports to the family doctor Dr. Wong on a visit that John has been frequently anxious and cannot get into sleep at night for more than 6 months. Dr. Wong decides to prescribe a short course of diazepam to him. How does diazepam work? A. Activating GABAB receptors in the central nervous system. B. Blocking glutamate receptors in hierarchical neuronal pathways in the brain C. Increasing the frequency of Cl- channel opening which is coupled to GABAA receptors D. Inhibiting GABA-transaminase leading to increased levels of GABA Ans: C ________________ HNS36 Autonomic nervous system / ________________ HNS44 Antidepressants [2020 M24 2nd Summative Q55] What is the mechanism of action of selective serotonin reuptake inhibitors as antidepressants in the treatment of depression? A. It enhances the release of serotonin at the postsynaptic cell. B. It enhances the release of serotonin at the presynaptic cell. C. It limits the reabsorption of serotonin into the postsynaptic cell. D. It limits the reabsorption of serotonin into the presynaptic cell. Ans: D [2021 M25 2nd Summative Q50] Depression is caused by low levels or a deficit in function of neurotransmitters. Which of the following is the hypothesis for the pathogenesis of depression? A. Dopamine hypothesis B. Melatonin hypothesis C. Monoamine hypothesis D. Tricyclic hypothesis Ans: C [2022 M26 2nd Summative Q55] A low level or a deficit in the function of neurotransmitters is suggested to be the cause of depression. What is the mode of action of noradrenaline (NA) reuptake inhibitor on treating depression? A. It blocks the α2 adrenoceptor on the presynapse to prevent the reuptake of NA. B. It blocks the NA receptor on the postsynapse to prevent the reuptake of NA. C. It blocks the NA reuptake transporter on the presynapse to reduce the reuptake of NA. D. It inhibits the NA reuptake transporter on the postsynapse to prevent the negative feedback loop and increase the release of NA. Ans: C Explanation / reference to students: On the presynaptic terminals, there are noradrenaline transporters that can take up the noradrenaline in the synapses. Noradrenaline (NA) reuptake inhibitor is the drug that can directly act on this noradrenaline transporters to prevent the uptake of noradrenaline. Do not mix it up with alpha-2 adrenoceptors, which can suppress the release of noradrenaline while these receptors are stimulated. Both alpha-2 adrenoceptor inhibitor and noradrenaline reuptake inhibitor can increase noradrenaline level in the synapse, but their mechanisms are different. ________________ [2023 M27 2nd Summative Q50] A low level or a deficit in the function of neurotransmitters is suggested to be the cause of depression. Which of the following is the hypothesis for the pathogenesis of depression? A. Glutamate hypothesis B. Melatonin hypothesis C. Neurotrophic hypothesis D. Serotonin hypothesis Ans: C [2023 M27 2nd Summative Q51] Monoamine hypothesis suggests that depression is caused by low levels or a deficit in function of neurotransmitters. What is the mode of action of a2 adrenoceptor antagonist (NASA) in treating depression? A. It induces the synthesis of the neurotransmitter. B. It prevents the binding of the neurotransmitter to their autoreceptors on presynapse. C. It prevents the degradation of the neurotransmitter. D. It prevents the reuptake of neurotransmitters by the transporters on the post-synapse. Ans: B [2024 M28 Sample Questions] A low level or a deficit in the function of neurotransmitters is suggested to be the cause of depression. What is the mode of action of noradrenaline (NA) reuptake inhibitor on treating depression? A. It blocks the ɑ2 adrenoceptor on the presynaptic cell to prevent the reuptake of NA. B. It blocks the NA receptor on the postsynaptic cell to prevent the reuptake of NA. C. It blocks the NA reuptake transporter on the presynaptic cell to reduce the reuptake of NA. D. It inhibits the NA reuptake transporter on the postsynaptic cell to prevent the negative feedback loop and increase the release of NA. Ans: C Explanation / reference to students: On the presynaptic terminals, there are noradrenaline transporters that can take up the noradrenaline in the synapses. Noradrenaline (NA) reuptake inhibitor is the drug that can directly act on this noradrenaline transporters to prevent the uptake of noradrenaline. Do not mix it up with alpha-2 adrenoceptors, which can suppress the release of noradrenaline while these receptors are stimulated. Both alpha-2 adrenoceptor inhibitor and noradrenaline reuptake inhibitor can increase noradrenaline level in the synapse, but their mechanisms are different. ________________ HNS45 Antipsychotics [2019 M23 2nd Summative Q54] Dopamine receptors are widely expressed in different regions of the brain. Blockade of dopamine D2 receptors in the mesocortical and mesolimbic region of the brain is associated with antipsychotic efficacy. Which of the following is the major side effect of blocking the dopamine D2 receptors in the tuberoinfundibular region of the brain? A. Acute dystonia B. Akathisia C. Gynecomastia D. Increased risk of diabetes E. Schizophrenia Ans: C [2020 M24 2nd Summative Q50] Blockade of dopamine D2 receptors in different regions of the brain is associated with various side effects. What is the major side effect of anti-psychotics on blocking the dopamine receptor 2 receptors on the nigrostriatal region of the brain? A. Galactorrhoea B. Increased risk of diabetes C. Manifestation of extrapyramidal symptoms D. Sexual dysfunction Ans: C ________________ HNS50 Drugs used for neurodegenerative diseases [2022 M26 2nd Summative Q56] Mrs. Chao was an 83-year-old woman when she was diagnosed with early Alzheimer's disease (AD). Rivastigmine patch was being prescribed. Another 6 years have passed and Mrs. Chao's functional abilities (activities of daily living, communication and memory) have significantly declined. During her recent routine check-up, which of the following drugs would MOST LIKELY be added by the doctor? A. Amantadine B. Benztropine C. Bromocriptine D. Memantine Ans: D ________________ Biochemistry ________________ HNS08 Chemical neurotransmission I [2019 M23 2nd Summative Q44] Calcium-triggered neurotransmitter release requires specific docking of synaptic vesicles to target membrane proteins at the presynaptic terminal. Which presynaptic protein is a proteolytic target of botulinum toxins? A. Ca2+/calmodulin dependent protein kinase B. Clathrin C. Synaptotagmin D. Syntaxin E. Voltage-gated Ca2+ channel protein Ans: D Botulinum toxins: BoTX B, D, F, G cleave synaptobrevin (V-SNARE protein); BoTX C cleaves syntaxin (T-SNARE on presynaptic plasma membrane), BoTX A, E cleave SNAP-25 (T-SNARE on presynaptic plasma membrane) [2021 M25 2nd Summative Q44] A 5-year-old with myasthenic symptoms and episodic apnoea since birth indicated no decrement at baseline response on 2-Hz repeated stimulation of motor nerves (RNS). 10-Hz RNS for 5 minutes however triggered (i) decremental response on 2-Hz RNS, (ii) drop in compound muscle action potential (CMAP) amplitude, and (iii) subsequent post tetanic recovery of CMAP amplitude for up to 30 minutes. What synaptic protein is MOST LIKELY the causative molecular defect in this patient? A. Acetylcholine receptor (AChR) B. Choline acetyltransferase (ChAT) C. High-affinity presynaptic choline transporter (ChT) D. Synaptosomal-associated protein, 25 kDa (SNAP25) Ans: B Comment from Question Writer * The topic was covered in lecture HNS06, specifically in ppt slides 21 – 22 under the context of “Presynaptic targets of neurological disorders”. Congenital myasthenic syndrome (CMS) with episodic apnea was highlighted as one such disorder and the most common among presynaptic CMSs. Presynaptic CMSs can be caused by defects in proteins involved in (i) synthesis of the neurotransmitter, acetylcholine, (ii) packaging of acetylcholine into synaptic vesicles, (iii) vesicle docking at the presynaptic nerve terminal, and (iv) release of acetylcholine into the synaptic cleft following vesicle fusion with the presynaptic nerve terminal. * Among the 4 options listed in the question, * A. Acetylcholine receptor (AChR) is the postsynaptic receptor of the neurotransmitter, ACh, at the neuromuscular junction. Defect in AChR is due to autoimmune attack, often with adult onset. Electro-diagnostic testing is expected to yield decremental response to 2 Hz RNS and increased jitter/blocking upon single fibre electromyography. [ None of these features are indicated in the Q. (A) is therefore NOT likely the causative defect.] * B. Choline acetyltransferase (ChAT) catalyzes the synthesis of ACh from choline and acetyl CoA in the presynaptic nerve terminal. Recessive mutations in the gene for ChAT cause reduction in enzyme activity, causing apneic spells as early as infancy but minimal or no myasthenic symptoms between spells. Between episodic exacerbations, patients possibly display no decremental response on baseline/2-Hz RNS, indicating that vesicular stores of ACh at the presynaptic nerve terminal were adequate to sustain the response. Conditioning with 10 Hz RNS for 5 min can trigger decremental response upon 2-Hz RNS and decline in CMAP, suggesting decline in vesicular stores of Ach with the conditioning train of 10-Hz stimuli. Subsequent slow recovery of CMAP amplitude further suggests defect in acetylcholine resynthesis. [These features agree with those indicated in the Q. (B) is likely the causative defect.] * C. High-affinity presynaptic choline transporter (ChT). Cleavage of ACh by acetylcholinesterase in the synaptic cleft yields choline and decline in cholinergic neurotransmission follows. Following uptake via ChT at the presynaptic nerve terminal, choline is used for re-synthesis of ACh. Recessive mutations in the gene for ChT causes congenital myasthenic syndrome (CMS) with episodic apnoea as early as infancy. Decremental response even to low frequency RNS or after few seconds of conditioning with high-frequency (20 – 50 Hz) RNS. [This differs from features described in the Q. (C) is therefore NOT likely the causative defect.] * D. Synaptosomal-associated protein, 25 kDa (SNAP25). SNAP25 deficiency abolishes vesicle priming and fast calcium-triggered vesicular release of neurotransmitter. Mutation in SNAP25 caused manifestation of symptoms even in utero. Decremental response was observed even to low-frequency RNS; responses to high-frequency RNS or facilitation upon tetanic stimulation were not reported. [These features differ from those described in the Q. (D) is therefore NOT likely the causative defect.] [2022 M26 2nd Summative Q51] Congenital myasthenic syndromes (CMS) are autosomal recessive disorders that affect quantal acetylcholine release into the neuromuscular junction (NMJ). What is a viable means to maintain acetylcholine level in the NMJ for neurotransmission in patients with CMS? A. Increase choline uptake at the NMJ B. Increase dietary intake of choline C. Inhibit acetylcholinesterase activity in the NMJ D. Stimulate choline acetyltransferase activity at the nerve terminal Ans: C ________________ HNS09 Chemical neurotransmission II [2019 M23 2nd Summative Q45] Neurotransmitter-gated ion channels, also known as ionotropic receptors, at the post-synapses are responsible for fast synaptic transmission. What neurotransmitter mediates increased flux of cations through the channels, directly causing excitatory transmission? A. Gamma-aminobutyric acid B. Glutamic acid C. Glycine D. Kainic acid E. N-methyl-D-aspartic acid Ans: B A: Incorrect, GABA is the primary inhibitory neurotransmitter in the central nervous system B: Correct, Glutamic acid = glutamate → excitatory C: Incorrect, Glycine is an inhibitory neurotransmitter found mainly in the spinal cord and brainstem D: Incorrect, Kainic acid is an agonist of glutamate receptors, but not the neurotransmitter itself E: Incorrect, NMDA is a specific agonist of a subtype of glutamate receptors, but not the neurotransmitter [2020 M24 2nd Summative Q43] Neurotransmitter-gated ion channels, also known as ionotropic receptors, at the post-synapse are responsible for fast synaptic transmission. What amino acid functions as such a neurotransmitter, mediating flux of cations through the channels and directly causing excitatory transmission? A. Gamma-aminobutyric acid B. Glutamic acid C. Glycine D. N-methyl-D-aspartic acid Ans: B ________________ Molecular medicine ________________ HNS48 Neurodegenerative diseases [2023 M27 2nd Summative Q45] Magnetic resonance imaging (MRI) can assess structural atrophy in the early state of Alzheimer disease. Which of the following brain structures displays atrophy in the mild cognitive impairment of Alzheimer disease by MRI assessment? A. Inferior frontal gyrus B. Lingual gyrus C. Parahippocampal gyrus D. Supramarginal gyrus Ans: C ________________ HNS49 Molecular basis of neurological diseases [2020 M24 2nd Summative Q42] One observable change in the brain of Parkinson disease patients is the loss of neuromelanin (depigmentation) in substantia nigra. From which of the following is neuromelanin derived? A. Alpha-synuclein B. Dopamine C. Melatonin D. Parkin Ans: B ________________ Public health ________________ HNS51 When we all become Methuselah [2022 M26 2nd Summative Q61] An increasing life expectancy at birth represents one of the two MOST IMPORTANT factors driving global population ageing. What is the second factor? A. Decreasing age-standardised mortality rate B. Decreasing fertility rate C. Decreasing infant mortality rate D. Increasing prevalence of infertility among women of child bearing age Ans: B [2023 M27 2nd Summative Q56] Functional capacity such as cardiac output, ventilatory capacity, and muscle strength generally peaks in early adulthood and declines afterwards. Exposure to various behavioural and environmental factors during which period of life may affect the rate of decline in functional capacity? A. Early adulthood B. Early childhood C. Elderly years D. Over the whole life course Ans: D ________________ HNS52 Impacts of evidence-based versus defensive medical practices: To CT or not to CT in traumatic head injury [2021 M25 2nd Summative Q56] The incidence of traumatic brain injury in paediatric age group varies from birth to teenage years. In which gender(s) does/do this age-dependent incidence follow a positively skewed pattern? A. In both genders B. In females C. In males D. In neither gender Ans: B Comment from Question Writer * The examined issue was explicitly covered in exactly the same wording, which has also been mentioned in the audio recording of the WCS. The issue had also been reiterated in the later interactive session. * ________________ Others [2024 M28 Sample Questions] A patient has experienced a sudden and complete interruption of his cerebral blood supply. Which of the following changes are MOST LIKELY present? A. Increased number of rapid waves on electroencephalography (EEG) B. Increased release of excitatory neurotransmitters C. Reduced calcium influx into neurons D. Reduced lactate level within neurons Ans: B Explanation / reference to students: Option A: EEG will slow down in ischaemia. Option B: Correct – such as glutamate. Option C: Increased calcium influx is one of the mechanisms causing ischaemic neuronal injury. Option D: Intracellular lactic acidosis may occur in ischaemia. [2024 M28 Sample Questions] A constant supply of oxygenated blood is essential for brain functioning and survival. Which of the following would MOST LIKELY be present in acute severe brain ischaemia? A. Calcium efflux from neurons B. Increased cell membrane Na+/K+-ATPase activity C. Increased release of glutamate by neurons D. Intracellular alkalosis of neurons Ans: C Explanation / reference to students: Option A: There is increased calcium influx into neurons. Option B: Hypoxia will slow down ATPase activity. Option C: Correct - increased release of excitatory neurotransmitters, such as glutamate, is a hallmark of ischaemic neuronal injury. Option D: In hypoxia, pyruvate is metabolised into lactate, causing intracellular lactic acidosis.